Difference between revisions of "1968 AHSME Problems/Problem 29"

(Created page with "== Problem == Given the three numbers <math>x,y=x^x,z=x^{x^x}</math> with <math>.9<x<1.0</math>. Arranged in order of increasing magnitude, they are: <math>\text{(A) } x,z,y\qu...")
 
m (Solution)
Line 10: Line 10:
  
 
== Solution ==
 
== Solution ==
<math>\fbox{}</math>
+
<math>\fbox{A}</math>
  
 
== See also ==
 
== See also ==

Revision as of 03:34, 29 September 2014

Problem

Given the three numbers $x,y=x^x,z=x^{x^x}$ with $.9<x<1.0$. Arranged in order of increasing magnitude, they are:

$\text{(A) } x,z,y\quad \text{(B) } x,y,z\quad \text{(C) } y,x,z\quad \text{(D) } y,z,x\quad \text{(E) } z,x,y$

Solution

$\fbox{A}$

See also

1968 AHSME (ProblemsAnswer KeyResources)
Preceded by
Problem 28
Followed by
Problem 30
1 2 3 4 5 6 7 8 9 10 11 12 13 14 15 16 17 18 19 20 21 22 23 24 25 26 27 28 29 30
All AHSME Problems and Solutions

The problems on this page are copyrighted by the Mathematical Association of America's American Mathematics Competitions. AMC logo.png